Difference between revisions of "2021 Fall AMC 10B Problems/Problem 2"

m
(Tag: New redirect)
 
Line 1: Line 1:
==Problem==
+
#REDIRECT [[2021_Fall_AMC_12B_Problems/Problem_2]]
 
 
What is the area of the shaded figure shown below?
 
<asy>
 
size(200);
 
defaultpen(linewidth(0.4)+fontsize(12));
 
pen s = linewidth(0.8)+fontsize(8);
 
 
 
pair O,X,Y;
 
O = origin;
 
X = (6,0);
 
Y = (0,5);
 
fill((1,0)--(3,5)--(5,0)--(3,2)--cycle, palegray+opacity(0.2));
 
for (int i=1; i<7; ++i)
 
{
 
draw((i,0)--(i,5), gray+dashed);
 
label("${"+string(i)+"}$", (i,0), 2*S);
 
if (i<6)
 
{
 
draw((0,i)--(6,i), gray+dashed);
 
label("${"+string(i)+"}$", (0,i), 2*W);
 
}
 
}
 
label("$0$", O, 2*SW);
 
draw(O--X+(0.15,0), EndArrow);
 
draw(O--Y+(0,0.15), EndArrow);
 
draw((1,0)--(3,5)--(5,0)--(3,2)--(1,0), black+1.5);
 
</asy>
 
 
 
<math>(\textbf{A})\: 4\qquad(\textbf{B}) \: 6\qquad(\textbf{C}) \: 8\qquad(\textbf{D}) \: 10\qquad(\textbf{E}) \: 12</math>
 
 
 
==Solution #1==
 
 
 
We have <math>2</math> isosceles triangles. Thus, the area of the shaded region is <math>\frac{1}{2} \cdot 5 \cdot 4 - \left(\frac{1}{2} \cdot 4 \cdot 2\right) = 10 - 4 = 6.</math> Thus our answer is <math>\boxed{(\textbf{B})}.</math> 
 
 
 
~NH14
 
 
 
 
 
==Solution #2==
 
 
 
As we can see, the shape is symmetrical, so it will be equally valid to simply calculate one of the half's area and multiply by 2. One half's area is <math>\frac{bh}2</math>, so two halves would be <math>bh=3\cdot2=6</math>. Thus our answer is <math>\boxed{(\textbf{B})}.</math>
 
 
 
~Hefei417, or 陆畅 Sunny from China
 
 
 
 
 
==Solution #3 (Overkill)==
 
We start by finding the points. The outlined shape is made up of <math>(1,0),(3,5),(5,0),(3,2)</math>. By the
 
Shoelace Theorem, we find the area to be <math>6</math>, or <math>\boxed{B}</math>.
 
 
 
 
 
https://artofproblemsolving.com/wiki/index.php/Shoelace_Theorem
 
 
 
~Taco12
 
 
 
~I-AM-DA-KING for the link
 
 
 
==Solution #4 (Pick's Theorem)==
 
 
 
We can use Pick's Theorem. We have <math>4</math> interior points and <math>6</math> boundary points. By Pick's Theorem, we get <math>4+\frac{6}{2}-1 = 4+3-1 = 6.</math> Checking our answer choices, we find our answer to be <math>\boxed{B}</math>.
 
 
 
~danprathab
 
 
 
== Solution 5 ==
 
The area is
 
<cmath>
 
\begin{align*}
 
\frac{1}{2} \left( 5 - 1 \right) 5 - \frac{1}{2} \left( 5 - 1 \right) 2
 
& = 6 .
 
\end{align*}
 
</cmath>
 
 
 
Therefore, the answer is <math>\boxed{\textbf{(B) }6}</math>.
 
 
 
~Steven Chen (www.professorchenedu.com)
 
== Solution 6 ==
 
We have that the area of the shaded region is the difference between the area of 2 triangles. So, we have the area is <math>\dfrac{1}{2}(4\cdot 5) - \dfrac{1}{2}(4\cdot 2) = \boxed{\textbf{(B) }6}</math> ~~stjwyl
 
==Video Solution by Interstigation==
 
https://youtu.be/p9_RH4s-kBA?t=110
 
 
 
==See Also==
 
{{AMC10 box|year=2021 Fall|ab=B|num-a=3|num-b=1}}
 
{{MAA Notice}}
 

Latest revision as of 06:45, 3 January 2022